Quantcast
  • Register
PhysicsOverflow is a next-generation academic platform for physicists and astronomers, including a community peer review system and a postgraduate-level discussion forum analogous to MathOverflow.

Welcome to PhysicsOverflow! PhysicsOverflow is an open platform for community peer review and graduate-level Physics discussion.

Please help promote PhysicsOverflow ads elsewhere if you like it.

News

PO is now at the Physics Department of Bielefeld University!

New printer friendly PO pages!

Migration to Bielefeld University was successful!

Please vote for this year's PhysicsOverflow ads!

Please do help out in categorising submissions. Submit a paper to PhysicsOverflow!

... see more

Tools for paper authors

Submit paper
Claim Paper Authorship

Tools for SE users

Search User
Reclaim SE Account
Request Account Merger
Nativise imported posts
Claim post (deleted users)
Import SE post

Users whose questions have been imported from Physics Stack Exchange, Theoretical Physics Stack Exchange, or any other Stack Exchange site are kindly requested to reclaim their account and not to register as a new user.

Public \(\beta\) tools

Report a bug with a feature
Request a new functionality
404 page design
Send feedback

Attributions

(propose a free ad)

Site Statistics

205 submissions , 163 unreviewed
5,047 questions , 2,200 unanswered
5,345 answers , 22,709 comments
1,470 users with positive rep
816 active unimported users
More ...

  Why not formulate Quantum Mechanics using Lagrangians?

+ 6 like - 0 dislike
1712 views

As the title implies, why is it that the most common formalisms we use in quantum mechanics prefer to describe systems in the terms of a Hamiltionian instead of a Lagrangian?

Is there some convenience to defining our systems one way over the other? Are there cases I'm not aware of where Lagrangian formalism is preferred?

This post imported from StackExchange Physics at 2014-04-05 04:38 (UCT), posted by SE-user sakanojo
asked Nov 22, 2013 in Theoretical Physics by sakanojo (30 points) [ no revision ]
Would be interesting for someone to give a pro argument for using Lagrangians.

This post imported from StackExchange Physics at 2014-04-05 04:38 (UCT), posted by SE-user BMS
Path integrals.

This post imported from StackExchange Physics at 2014-04-05 04:38 (UCT), posted by SE-user Dimensio1n0
Possible duplicate: physics.stackexchange.com/q/21866/2451 and links therein.

This post imported from StackExchange Physics at 2014-04-05 04:38 (UCT), posted by SE-user Qmechanic
Variational formulation.

This post imported from StackExchange Physics at 2014-04-05 04:38 (UCT), posted by SE-user Dimensio1n0

3 Answers

+ 4 like - 0 dislike

It's because they're based on the historical approach: Schroedinger's equation.

Schroedinger's equation was discovered on its own before we knew about canonical quantization. Dirac came up with the canonical quantization rules which re-wrote (and generalized) Schroedinger's equation into the familiar one we have today, $\hat{H} \psi = i \dot{\psi}$.

That said, there is an approach which uses the action (and thus the Lagrangian or Lagrangian density) due to Feynman: The path integral approach. This approach has as its biggest advantage the ability to be reconciled with Special Relativity, which proved much too difficult a task for extensions of the Schroedinger equation (the Dirac Equation was the most successful attempt, but wasn't general enough to describe some phenomena).

This is what is used in the most advanced quantum physics such as quantum field theory, quantum electrodynamics being the best example. But unless you're interested in high-energy particle physics or really advanced condensed matter physics, the traditional quantum mechanics is sufficient.

This post imported from StackExchange Physics at 2014-04-05 04:38 (UCT), posted by SE-user ghollisjr
answered Nov 22, 2013 by ghollisjr (40 points) [ no revision ]
+ 2 like - 0 dislike

I don't have an answer for why there is no simple Lagrangian formulation, but I can explain some of why a Hamiltonian one is easy. Part of the way to go from Classical Mechanics to Quantum is by replacing Poisson brackets with commutators, and observables with operators on Hilbert space and their expectation values. So the equation

$\frac{d}{dt} f(q, p, t) = \left\{ f,H \right\} + \frac{\partial f}{\partial t} $

becomes the quantum

$\frac{d}{dt} \langle f \rangle = -i \langle\left[f,H \right]\rangle + \langle \frac{\partial f}{\partial t} \rangle.$

So the Hamiltonian is convenient because it gives the time evolution of operators, states, and expectation values directly. Also, because the Hamiltonian is a conserved quantity, stationary states (i.e. those that do not evolve in time) will be eigenvectors of the Hamiltonian, and eigenvalue problems are easy.

This post imported from StackExchange Physics at 2014-04-05 04:38 (UCT), posted by SE-user ZachMcDargh
answered Nov 22, 2013 by ZachMcDargh (20 points) [ no revision ]
This answer is much closer to what I'm looking for! Isn't the Lagrangian also a conserved quantity whenever the Hamiltonian is conserved? Or am I not remembering the Lagrangian correctly?

This post imported from StackExchange Physics at 2014-04-05 04:38 (UCT), posted by SE-user sakanojo
If the lagrangian does not depend explicitly on time, then the hamiltonian is conserved. You can tell the lagrangian because the potential energy goes down, the kinetic term goes up, both of which increase the lagrangian.

This post imported from StackExchange Physics at 2014-04-05 04:38 (UCT), posted by SE-user ZachMcDargh
+ 0 like - 0 dislike

I can think of several reasons for why using Hamiltonians is preferred, but the most important, I'ld say, is that you need to use path integral formalism in order to formulate (non relativistic) QM in terms of the Lagrangian, which, for an undergrad course, is a bit of an overkill.

Also, many of the most renowned equations in QM like, say, the Schrödinger Equation, use the Hamiltonian: $\hat H \Psi=\hat E \Psi$ Thus, although it is possible, why change it? It would be quite a pain to do so.

For what I understand, however, modern QM relies heavily on both the Hamiltonian and the Lagrangian formalism.

Hope it helped!

This post imported from StackExchange Physics at 2014-04-05 04:38 (UCT), posted by SE-user Demian Licht
answered Nov 22, 2013 by Demian Licht (0 points) [ no revision ]
Thanks for offering some thoughts on the matter! Overkill for an undergrad course is unfortunately not much of a rationale. As for the the Schrödinger Equation, is there a particular reason why using Hamiltonian formulation results in something 'cleaner' or more usable than Lagrangian?

This post imported from StackExchange Physics at 2014-04-05 04:38 (UCT), posted by SE-user sakanojo
Sorry for not being much more rational, but I wasn't sure what type of answer you where looking for :p. Anyways, yes, there are several reasons as to why use the Hamiltonian, first of all it provides an answer in terms of momentum, and allows us to predict further onto the future, also, it is derived from a Lagrangian so it can be reversed back into one. As for the Schrödinger Equation, the Hamiltonian explains the time-evolution of the wave function, in terms of plank's and i, that is, which is clearly advantageous when dealing with the evolution of a system.

This post imported from StackExchange Physics at 2014-04-05 04:38 (UCT), posted by SE-user Demian Licht
Not a problem! I am looking for something more specific, however. Lagrangian formalism provides a perfect framework for examining the evolution of a system in time with classical systems - why would a Hamiltonian allow us to better see into the future than a Lagrangian? Sadly, 'clearly advantageous' isn't much of an rationale. Clearly advantageous as opposed to what?

This post imported from StackExchange Physics at 2014-04-05 04:38 (UCT), posted by SE-user sakanojo
Ok, I think I get to where you're going, my best explanation would be that the Hamiltonian is easier to diagonalize for it has less degrees of freed, sadly I am not capable of fully explaining this over the a thread, you could try to access this page: mathpages.com/home/kmath523/kmath523.htm or contact me, I would be more than happy to help you!

This post imported from StackExchange Physics at 2014-04-05 04:38 (UCT), posted by SE-user Demian Licht
I'll take a look now! Thanks for the link.

This post imported from StackExchange Physics at 2014-04-05 04:38 (UCT), posted by SE-user sakanojo

Your answer

Please use answers only to (at least partly) answer questions. To comment, discuss, or ask for clarification, leave a comment instead.
To mask links under text, please type your text, highlight it, and click the "link" button. You can then enter your link URL.
Please consult the FAQ for as to how to format your post.
This is the answer box; if you want to write a comment instead, please use the 'add comment' button.
Live preview (may slow down editor)   Preview
Your name to display (optional):
Privacy: Your email address will only be used for sending these notifications.
Anti-spam verification:
If you are a human please identify the position of the character covered by the symbol $\varnothing$ in the following word:
p$\hbar$ysicsOverfl$\varnothing$w
Then drag the red bullet below over the corresponding character of our banner. When you drop it there, the bullet changes to green (on slow internet connections after a few seconds).
Please complete the anti-spam verification




user contributions licensed under cc by-sa 3.0 with attribution required

Your rights
...